Ανισότητα!

Συντονιστές: achilleas, emouroukos, silouan

Άβαταρ μέλους
Ορέστης Λιγνός
Δημοσιεύσεις: 1835
Εγγραφή: Κυρ Μάιος 08, 2016 7:19 pm
Τοποθεσία: Χαλάνδρι Αττικής
Επικοινωνία:

Ανισότητα!

#1

Μη αναγνωσμένη δημοσίευση από Ορέστης Λιγνός » Δευ Ιαν 01, 2018 4:44 pm

Χρόνια Πολλά και Καλή χρονιά σε όλους!

Αν a,b,c>0, δείξτε ότι \dfrac{1}{a}+\dfrac{1}{b}+\dfrac{1}{c} \leqslant \dfrac{a^8+b^8+c^8}{a^3b^3c^3}.


Κερδίζουμε ό,τι τολμούμε!

Λέξεις Κλειδιά:
Άβαταρ μέλους
Demetres
Γενικός Συντονιστής
Δημοσιεύσεις: 8989
Εγγραφή: Δευ Ιαν 19, 2009 5:16 pm
Τοποθεσία: Λεμεσός/Πύλα
Επικοινωνία:

Re: Ανισότητα!

#2

Μη αναγνωσμένη δημοσίευση από Demetres » Δευ Ιαν 01, 2018 5:16 pm

Καλή χρονιά Ορέστη!

Από ΑΜ-ΓΜ έχουμε:

3a^8 + 3b^8 + 2c^8 = a^8+a^8+a^8+b^8+b^8+b^8+c^8+c^8 \geqslant 8\sqrt[8]{(a^8)^3(b^8)^3(c^8)^2} = 8a^3b^3c^2

και ομοίως

3a^8 + 2b^8 + 3c^8 \geqslant 8a^3b^2c^3

και

2a^8 + 3b^8 + 3c^8 \geqslant 8a^2b^3c^3

Προσθέτοντας και διαιρώντας με 8a^3b^3c^3 λαμβάνουμε το ζητούμενο.


Datis-Kalali
Δημοσιεύσεις: 117
Εγγραφή: Δευ Δεκ 12, 2016 5:33 pm
Τοποθεσία: Λευκωσία

Re: Ανισότητα!

#3

Μη αναγνωσμένη δημοσίευση από Datis-Kalali » Δευ Ιαν 01, 2018 5:22 pm

Ορέστης Λιγνός έγραψε:
Δευ Ιαν 01, 2018 4:44 pm
Χρόνια Πολλά και Καλή χρονιά σε όλους!

Αν a,b,c>0, δείξτε ότι \dfrac{1}{a}+\dfrac{1}{b}+\dfrac{1}{c} \leqslant \dfrac{a^8+b^8+c^8}{a^3b^3c^3}.
Χρόνια πολλά.

Η ανισότητα γράφεται:
a^8+b^8+c^8 \ge a^2b^3c^3+a^3b^2c^3+a^3b^3c^2
Τωρά απο την ανισότητα ΑΜ-ΓΜ έχουμε
a^8+a^8+b^8+b^8+b^8+c^8+c^8+c^8 \ge 8 (a^{16} b^{24} c^{24})^{\frac{1}{8}}=8a^2b^3c^3 \Rightarrow 2a^8+3b^8+3c^8 \ge 8a^2b^3c^3
Όμοια παιρνουμε:
2a^8+3b^8+3c^8 \ge 8a^2b^3c^3
2a^8+3b^8+3c^8 \ge 8a^2b^3c^3
Προσθέτοντας κατα μέλη και έχουμε
8a^8+8b^8+8c^8 \ge 8a^2b^3c^3+8a^3b^2c^3+8a^3b^3c^2
Άρα
a^8+b^8+c^8 \ge a^2b^3c^3+a^3b^2c^3+a^3b^3c^2 , δηλαδή το ζητούμενο.
Η ισότητα ισχυεί αν και μονον αν a=b=c


Άβαταρ μέλους
Διονύσιος Αδαμόπουλος
Δημοσιεύσεις: 807
Εγγραφή: Σάβ Μαρ 19, 2016 5:11 pm
Τοποθεσία: Πύργος Ηλείας

Re: Ανισότητα!

#4

Μη αναγνωσμένη δημοσίευση από Διονύσιος Αδαμόπουλος » Δευ Ιαν 01, 2018 5:23 pm

Καλή Χρονιά!

Η ανισότητα γίνεται:

a^8+b^8+c^8\geq a^3b^3c^2+a^3b^2c^3+a^2b^3c^3 που ισχύει από Muirhead, καθώς (8,0, 0)>(3, 3, 2).


Houston, we have a problem!
Datis-Kalali
Δημοσιεύσεις: 117
Εγγραφή: Δευ Δεκ 12, 2016 5:33 pm
Τοποθεσία: Λευκωσία

Re: Ανισότητα!

#5

Μη αναγνωσμένη δημοσίευση από Datis-Kalali » Δευ Ιαν 01, 2018 5:35 pm

Ορέστης Λιγνός έγραψε:
Δευ Ιαν 01, 2018 4:44 pm
Χρόνια Πολλά και Καλή χρονιά σε όλους!

Αν a,b,c>0, δείξτε ότι \dfrac{1}{a}+\dfrac{1}{b}+\dfrac{1}{c} \leqslant \dfrac{a^8+b^8+c^8}{a^3b^3c^3}.

Πιο γενικά , για κάθε a,b,c>0 Να αποδείξετε ότι: \frac{1}{a^{3k-n}}+\frac{1}{b^{3k-n}}+\frac{1}{b^{3k-n}} \le \frac{a^n+b^n+c^n}{a^k b^k c^k}
Υπόδειξη : Βοηθά η ανισότητα Muirhead


ΠΑΠΑΔΟΠΟΥΛΟΣ ΣΤΑΥΡΟΣ
Δημοσιεύσεις: 3600
Εγγραφή: Πέμ Φεβ 27, 2014 9:05 am
Τοποθεσία: ΧΑΛΚΙΔΑ- ΑΘΗΝΑ-ΚΡΗΤΗ

Re: Ανισότητα!

#6

Μη αναγνωσμένη δημοσίευση από ΠΑΠΑΔΟΠΟΥΛΟΣ ΣΤΑΥΡΟΣ » Δευ Ιαν 01, 2018 8:51 pm

Datis-Kalali έγραψε:
Δευ Ιαν 01, 2018 5:35 pm
Ορέστης Λιγνός έγραψε:
Δευ Ιαν 01, 2018 4:44 pm
Χρόνια Πολλά και Καλή χρονιά σε όλους!

Αν a,b,c>0, δείξτε ότι \dfrac{1}{a}+\dfrac{1}{b}+\dfrac{1}{c} \leqslant \dfrac{a^8+b^8+c^8}{a^3b^3c^3}.

Πιο γενικά , για κάθε a,b,c>0 Να αποδείξετε ότι: \frac{1}{a^{3k-n}}+\frac{1}{b^{3k-n}}+\frac{1}{b^{3k-n}} \le \frac{a^n+b^n+c^n}{a^k b^k c^k}
Υπόδειξη : Βοηθά η ανισότητα Muirhead
Με την υπόθεση ότι n> 2k> k> 0


γράφεται a^{n-2k}b^{k}c^{k}+b^{n-2k}c^{k}a^{k}+c^{n-2k}a^{k}b^{k}\leq a^{n}+b^{n}+c^{n}


και είναι άμεση εφαρμογή της Holder με εκθέτες \frac{n}{n-2k},\frac{n}{k},\frac{n}{k}


ΤΣΟΠΕΛΑΣ ΓΙΑΝΝΗΣ
Δημοσιεύσεις: 298
Εγγραφή: Σάβ Απρ 03, 2010 5:06 pm
Τοποθεσία: Αμαλιάδα - Ηλείας

Re: Ανισότητα!

#7

Μη αναγνωσμένη δημοσίευση από ΤΣΟΠΕΛΑΣ ΓΙΑΝΝΗΣ » Δευ Ιαν 01, 2018 11:41 pm

Καλή χρονιά σε όλα τα μέλη του :santalogo:
μια ακόμη προσέγγιση...
a^{8}+b^{8}+c^{8}\geq a^{4}b^{4}+b^{4}c^{4}+c^{4}a^{4}=\left ( a^{2}b^{2} \right )^{2}+\left ( b^{2}c^{2} \right )^{2}+\left (c^{2}a^{2} \right )^{2}\geq a^{2}b^{2}b^{2}c^{2} +b^{2}c^{2}c^{2}a^{2}+c^{2}a^{2}a^{2}b^{2}=
\left ( ab^{2}c \right )^{2}+\left ( bc^{2}a \right )^{2}+\left (ca^{2}b \right )^{2}\geq ab^{2}cbc^{2}a+bc^{2}aca^{2}b+ca^{2}bab^{2}c=a^{2}b^{3}c^{3}+a^{3}b^{2}c^{3}+a^{3}b^{3}c^{2}
Δηλαδή με πολλαπλή χρήση της x^{2}+y^{2}+z^{2}\geq xy+yz+zx


Απάντηση

Επιστροφή σε “Άλγεβρα - Προχωρημένο Επίπεδο (Juniors)”

Μέλη σε σύνδεση

Μέλη σε αυτήν τη Δ. Συζήτηση: Δεν υπάρχουν εγγεγραμμένα μέλη και 2 επισκέπτες